Convergence of Series with Variable Exponent and Negative Power

  • Thread starter Thread starter mikethemike
  • Start date Start date
  • Tags Tags
    Series
Click For Summary
The discussion focuses on determining the convergence or divergence of the series Sum(n=1 to infinity) ((n+1)^(n-1))/(-n)^n. Initial attempts using the Ratio and Root tests yielded inconclusive results. Participants suggest applying the alternating series test and emphasize the importance of evaluating the limit and showing that the series is decreasing. One user recommends breaking the series into manageable components to identify a finite limit. The conversation highlights the challenges in proving convergence and the need for further analysis.
mikethemike
Messages
6
Reaction score
0

Homework Statement



Decide on the convergance/divergance of the following series:

Sum(n=1 to n= infinity) ((n+1)^(n-1))/(-n)^n

where ^ is to the power of and / is divided by.



2. The attempt at a solution

I've used both the Ratio and Root test which are inconclusive (ie. R=1, K=1). Tried changing it around to fit the Leibiniz criterion (and failed). I'm not sure where to go from here...


Thanks for any help :biggrin:

Mike
 
Physics news on Phys.org
Don't give up on the alternating series test. Take the absolute value. Can you show the limit is 0? Now try to show it's decreasing by showing the derivative of the log is negative.
 
Hey Dick,

I'm having trouble convincing myself that the limit is in fact, zero. I can't seem to prove this, even with the absolute value. Can you point me in the right direction?

Thanks

Mike
 
Sure. Break it into (n+1)^(n-1)/n^(n-1) times 1/n. The first factor has a finite limit. Can you find it?
 
Question: A clock's minute hand has length 4 and its hour hand has length 3. What is the distance between the tips at the moment when it is increasing most rapidly?(Putnam Exam Question) Answer: Making assumption that both the hands moves at constant angular velocities, the answer is ## \sqrt{7} .## But don't you think this assumption is somewhat doubtful and wrong?

Similar threads

  • · Replies 2 ·
Replies
2
Views
1K
  • · Replies 3 ·
Replies
3
Views
1K
  • · Replies 2 ·
Replies
2
Views
2K
  • · Replies 1 ·
Replies
1
Views
2K
  • · Replies 7 ·
Replies
7
Views
1K
  • · Replies 11 ·
Replies
11
Views
3K
  • · Replies 22 ·
Replies
22
Views
4K
  • · Replies 5 ·
Replies
5
Views
2K
Replies
14
Views
2K
Replies
7
Views
2K